Main Menu

Code cracking!

Started by Teyl Maktoyu Ayfìwopxä, October 30, 2011, 03:58:40 PM

Previous topic - Next topic

0 Members and 1 Guest are viewing this topic.

Teyl Maktoyu Ayfìwopxä

Nobody I asked so far can crack my code, who of you can?


#2 is 2 x #1
#3 is #2 + #4
#4 is #3 - #2

how much are #1, #2, #3 and #4?

A number can only be 1, 2, 3, 4, 5, 6, 7, 8 or 9...

+1 Karma for the first person!

Lance R. Casey

With the relationships given, #1 = #4 = 1, whereby #2 and #3 are freely variable as long as they meet the condition that #3 be one unit higher than #2. With the constraints in number assignment, the following solutions are possible:

1, 1, 2, 1
1, 2, 3, 1
1, 3, 4, 1
1, 4, 5, 1
1, 5, 6, 1
1, 6, 7, 1
1, 7, 8, 1
1, 8, 9, 1

// Lance R. Casey

Teyl Maktoyu Ayfìwopxä

Sorry, it had an error in it, I fixed it now - though I'll check if your codes are correct.

Lance R. Casey

Now it's even more loosely defined, since the last two equations are equivalent. The following values will all satisfy the two remaining conditions:

1, 1, 2, 1
1, 1, 3, 2
1, 1, 4, 3
1, 1, 5, 4
1, 1, 6, 5
1, 1, 7, 6
1, 1, 8, 7
1, 1, 9, 8
1, 2, 3, 1
1, 2, 4, 2
1, 2, 5, 3
1, 2, 6, 4
1, 2, 7, 5
1, 2, 8, 6
1, 2, 9, 7
1, 3, 4, 1
1, 3, 5, 2
1, 3, 6, 3
1, 3, 7, 4
1, 3, 8, 5
1, 3, 9, 6
1, 4, 5, 1
1, 4, 6, 2
1, 4, 7, 3
1, 4, 8, 4
1, 4, 9, 5
1, 5, 6, 1
1, 5, 7, 2
1, 5, 8, 3
1, 5, 9, 4
1, 6, 7, 1
1, 6, 8, 2
1, 6, 9, 3
1, 7, 8, 1
1, 7, 9, 2
1, 8, 9, 1

// Lance R. Casey

Teyl Maktoyu Ayfìwopxä

What about now?  :o

Do you have some program or whut?  ???

Lance R. Casey

The third equation is still redundant since it gives the exact same information as the second one. With #2 being twice the value of #1, and given the constraint to unit numbers, we get:

1, 2, 3, 1
1, 2, 4, 2
1, 2, 5, 3
1, 2, 6, 4
1, 2, 7, 5
1, 2, 8, 6
1, 2, 9, 7
2, 4, 5, 1
2, 4, 6, 2
2, 4, 7, 3
2, 4, 8, 4
2, 4, 9, 5
3, 6, 7, 1
3, 6, 8, 2
3, 6, 9, 3
4, 8, 9, 1


No program. ;)

// Lance R. Casey

bommel

yeah, two equations for four unknown values. This has either no solution at all or an unlimited amount. The constrains on the number will narrow the possible amount of solutions but mathematically there won't be a unique solution.

Teyl Maktoyu Ayfìwopxä

Hmm, didn't think about that yet, but my solution was 4, 8, 9, 1.

Thanks for the info though... O.O - there must be a riddle like that that I can create for this code?